Tutors’ report on the EQE 2023 papers and the meeting between tutors and EQE Committees

N. Cordes (NL), L. Ferreira (PT), A. Valborg Guðmundsdóttir (IS), A. Hards (DE), H. Marsman (NL), Z. Pintz (HU), S. van Rijnswou (NL), R. van Woudenberg (NL)


Each year in autumn, the EPO and the epi arrange a meeting of EQE tutors and members of the EQE Committees, usually referred to as “the Tutor Meeting”. The goals are to discuss last year’s papers, to improve future EQE’s by openly exchanging ideas and to help tutors prepare candidates for next year’s exam.

The Examination Board has kindly given the tutors permission to publish their own report of the important points so that candidates can more easily find this information. In addition, the comments can greatly assist when reading and interpreting the official Examiners’ Reports. The Tutors’ Report appears each year in the last edition of epi Information.

This year’s meeting was again held by videoconference, in the mornings of 18 and 19 October 2023. On the first day, General matters, the Pre-Exam and Papers A and D were discussed; on the second day, Papers B and C followed by Discussion, summary and closing.

On the first day, about 50-55 people participated of which about 35-40 tutors; on the second day, a few people less. The low attendance may be due to the online videoconference format, which is less attractive and less interactive than an onsite meeting, also due to the lack of social, informal moments. As expressed during the meeting by several tutors via video or the chat, an in-person or hybrid format is preferred and is essential to have the tutor meeting meet its goals. We all wish and look forward to having the 2024 Tutor Meeting again in person at the EPO in Munich or The Hague!

Some questions for the Committees were submitted via the EQE secretariat to the exam committees and the Examination Board by email prior to the meeting, unfortunately only by one or two tutors per exam paper. Unfortunately, these questions were not distributed to all participants, so that not all participants were aware about the questions asked and the arguments presented therewith. During the meeting, additional questions were asked by voice, video or chat. Most questions were addressed by the Examination Board and the Committees in the General part of the meeting or when discussing the papers. The answers are incorporated in this report and can be used to supplement the information from the Examiners’ Reports.

This Tutors’ Report contains the following sections:

  1. Pass rates EQE 2023;
  2. The Online EQE 2023;
  3. General remarks from the Tutor Meeting;
  4. Paper A;
  5. Paper B;
  6. Paper C;
  7. Paper D;
  8. Pre-Exam; and
  9. Concluding remarks.

On behalf of the tutors present in the meeting, I would like to thank all the members of the Examination Board and Committees as well as the EQE Secretariat for their openness, for listening to our opinions and comments, and for providing their feedback thereto. This meeting is our yearly opportunity to learn from each other. My thanks also go to the tutors who asked questions and contributed to the discussions.

My special thanks to my co-authors -in alphabetical order- Nico Cordes, Luis Ferreira, Anna Valborg Guðmundsdóttir, Andrew Hards, Harrie Marsman, Zsofia Pintz and Sander van Rijnswou for finding time to prepare the individual paper summaries.

We all wish you good luck in 2024,

Roel van Woudenberg (editor)

1) Pass rates EQE 2023

The official results for each paper of EQE 2023, as published on the EQE website and dated 18 July 2023 (as last year, one day after the candidates received their Results letters in MyEQE) for EQE2023, are shown in the table below:

Online EQE 2023* # Candidates PASS** COMP.FAIL FAIL***
Pre-Exam (4 x 70 min) 519 84,78% 15,22%
A (4 hours) 873 70,56% 4,35% 25,09%
B (3½ hours) 829 61,52% 9,17% 29,31%
C (2 x 3 hours) 1176**** 54,08% 12,08% 33,84%
D (27+28+45 marks; 1h45 + 1h40 + 2h45) 836 38,28% 10,292% 51,43%

* The Pre-Exam, A, B, C, and D papers are designed as papers of 4h, 3½h, 3h, 5h and 5h respectively [Rule 22-27 IPREE]. Since 2017, all candidates are granted an additional thirty minutes per paper to these durations [Decision of the Supervisory Board of 17 November 2016]. In the Online EQEs of 2021-2023, the Pre-Exam and C and D papers were split into multiple parts. Each part had to be completed before the start of the next break, with the next part only becoming available after the break. As a result, candidates were no longer free to allocate their time as they see fit across the different parts of the Pre-Exam and the C and D papers. To compensate for this restriction, the total duration of the Pre-Exam, C and D papers was extended to the lengths shown in this column. For paper D, the number of marks, number of questions and duration of each part varies from year to year [Notice Examination Board 13.03.2019]; this was the first D paper with a D2 part of only 45 marks (2022: 55; 2010: 50; 2010-2020: 60).

** note: These pass rates as published do not include the results of any appeals. It is not known whether/ how many appeals have been successful in interlocutory revision by the Examination Board or before the Disciplinary Board of Appeal or are still pending before the latter.

*** The FAIL rate includes no-shows.

**** The difference between the number of candidates sitting paper C compared to the numbers sitting A, B or C can be largely explained by the differences in pass rates in the previous two years: paper C had considerable lower pass rates than the other papers (usual for A and B; D was relatively high in 2021-2022 due to neutralizations of 25 resp 5 marks), so that there are considerably more resitters for paper C than for the other main exam papers this year.

In 2023, 519 candidates sat the Pre-Exam, which is somewhat less than in 2022 (680) and 2021 (626) and significantly less than in 2019 (920) and 2018 (935). This low number is probably caused by a relatively low number of resitters due to the very high pass rate in 2022 (97%; due to neutralization of part 3 of 25 marks and Q.20 of 5 marks) as well as due to a lower hiring of new patent attorney trainees during the COVID-19 pandemic. Of these 519 candidates, 440 (85%) passed the Pre-Exam. This pass-rate is comparable to that of the earlier papers which also contained two, concise cases/parts in the claims part (2021: 87% from 626 candidates; 2019: 88% from 920 candidates), and is somewhat higher than the typical pass rate for the 2015-2018 papers (74-76%; each 800-935 candidates) which each had a single and much longer claims part with much more reading.

690 out of 1630 candidates that took at least one paper passed the EQE (compared to 885 out of 1918 in 2022 and 1093 out of 2780 in 2021). When comparing the results for the individual main exam papers with earlier years:

  • The pass rate for Paper A was in the usual (wide) range (2022: 60%, 2021: 74%; 2019: 79%;
  • The pass rate for paper B was back to the usual range (2017: 67%, 2018 73%, 2019 53%, 2021 55%) after it having been quite high in 2022 (78%).
  • The pass rate for paper C was similar as in the years before (2022: 49%, 2021: 47%; 2019: 50%);
  • The pass rate of paper D was back in the usual range from before Covid (D 2019: 49%; 2018: 33%; 2017: 39%; 2016: 42%), after having been artificially high in 2021 and 2022 due to significant neutralizations (2021: complete D1-1 of 25 marks; 2022: one D1 question of 5 marks) .

2) The Online EQE 2023

In their “Information on the schedule for the EQE 2023 examination papers” communication of 27 September 2022, updated on 19 December 2022Information on the schedule for the EQE 2023 examination papers, 19 December 2022, it was indicated how the exam papers and the schedule would be adapted for the online EQE 2023. The document provided that:

“The EQE 2023 will take place online using the same setup as the EQE of 2021 and 2022 [comment from the editor: so, using the secure WISEflow/LockDown Browser]”.

“The pre-examination and both papers C and D will be split into parts. This means that candidates will not be free to allocate their time as they see fit across the different parts of the papers. To compensate for this restriction, the total duration of the relevant papers has been extended. Once the time allowed for a part has elapsed, it will not be possible to go back to that part.”

“The pre-examination will have the same syllabus and character as before, but it will be split into four parts. Each part must be completed before the start of the next break, with the next only becoming available after the break. The pre-examination lasts four hours and forty minutes. Candidates will be allowed to print the description of the invention as well as the prior-art documents for the claim analysis parts before the start of the appropriate part. The documents allowed for printing will be made available during the break preceding the relevant claim analysis part.”

Paper D will have the same syllabus and character as before, but it will be split into three parts. Each part must be completed before the start of the next break, with the next part only becoming available after the break. Paper D lasts six hours. N o calendars will be provided.”

Paper A will have the same syllabus and character as before [note from the editor: Paper A was not split in parts]. Paper A lasts four hours. Candidates will be allowed to print the prior-art documents and the drawings of the application, but not the letter of the applicant.”

Paper B will have the same syllabus and character as before [note from the editor: Paper B was not split in parts]. Paper B lasts 3.5 hours. Candidates will be allowed to print the prior-art documents and the drawing(s), but none of the following: the description and claims of the application, the EPO communication, the client’s letter and the amended claims.”

Paper C will have the same syllabus and character as before, but it will be split into two parts. The first part is to be completed before the break, with the second part only becoming available after the break. It will not be possible to go back to the first part after the break. Paper C lasts six hours.
Candidates will be allowed to print everything except the claims of the patent in suit/opposed.”

The communication also included the start and end times of each (part of the) paper, and information about possible unscheduled breaks. The latter were only allowed for paper A, B, C part 1, C part 2, and D2, but not for the shorter parts (the four Pre-Exam parts and the two D1 parts).

For A, B and C, the communication also indicated that “The documents allowed for printing will be made available approximately ten minutes before the start of the examination”. Printing was only available before entering the respective exam flow, but not anymore after entering the flow in the secure environment.

Compared to the 2021 and 2022 exams, the most significant changes in the schedule were the different lengths of the D1.1, D1.2 and D2 parts for paper D.

The EQE website also provided further documents such as the REE/IPREE (OJ EPO 2019, Suppl 2) “Announcement of the European qualifying examination 2023, OJ EPO 2022, A12”, the “Code of conduct for candidates during the EQE taking place online (e-EQE)” and “Instructions for answering the pre-examination paper and marking scheme” (Notice from the Examination Board dd 11 February 2021).

WISEflow

EQE 2023 was again conducted online using the locked browser in the examination platform WISEflow. WISEflow provides a secure online exam platform, allowing candidates to take the exam from any suitable location (without any other person in the room and without any other electronic equipment in the room/within reach apart from the computer and screen used for the exam), i.e., typically from home or the office. There were no examination centers. A computer with a network connection was required, and only a single screen could be used of a size and resolution at the candidate’s choice.

For the Pre-Exam 2023, WISEflow presents each question on the left half of the screen in a language selected from the three EPO language, as it did for Pre-Exam 2022. On the right half of the screen, 4 statements are presented in all three languages with a True/False answer option to each trilingual statement (clickable bullets). In the claims analysis parts, the prior art documents are provided as pdf documents via a hyperlink; these document could be printed before entering the exam flow and viewed online during the exam flow.
For the main exam papers, WISEflow provides a secure environment (FLOWlock) with the paper in pdf format and a proprietary editor with basic formatting functions (headers and ToC navigation pane; underline, bold, italics and strikethrough; enumerated lists, bullet lists). The editor allows a basic copy/paste from any text part of the examination paper into the editor, and within the editor. A basic form of annotation/ highlighting was available in the pdfs of the main exam papers; annotation/highlighting was also possible in the answer in the editor and in the Pre-Exam onscreen questions. We further refer to our report in epi Information 4/2022 on EQE 2022 and Wiseflow and its features.

As in the previous exams, candidates could bring any paper documentation, and make notes on paper. These notes could however not be handed in.

During the exam, the legal texts on the EPO website were also available in WISEflow via a hyperlink “Legal texts”, including the EPC, GL/EPO, GL/PCT-EPO, OJ EPO, National Law tables and the Euro-PCT Guide; as well as to some of the PCT legal texts on the WIPO website, in particular to the eGuide version of the PCT Applicant’s Guide. Candidates are recommended to check the situation in WISEflow during the Mocks exams as to which legal texts are available online and in which format. Wiseflow gave access to the version of the legal texts as in force in the date of the exam itself, rather than that on 31 October of the year before the exam as specified in Rules 2 and 22(1) IPREE as the relevant version; however, this had no effect on the answers and, as before, candidates are recommended to indicate which version they are using.

Mock exams/ Compendium

The EQE secretariat made part of the Compendium available in Wiseflow. These included all main exam 2022 and 2021 papers, A 2019, B 2019, C 2014, D 2016 and Pre-Exam 2019, 2021 and 2022 as well as mock main exam papers made by epi (the same mock papers as for e-EQE 2021). All enrolled candidates as well as any registered tutor had access to the Wiseflow Compendium.

The actual Online EQE 2023
The online exam took place from 7 – 16 March 2023 (Main Exam) and on 17 March 2023 (Pre-Exam). Compared to earlier years, the main exam papers were in the same sequences as before (D, A, B, C) with always at least one day in between two successive main exam papers.

3) General remarks from the Tutor Meeting

Opening words

The meeting itself opened with words of welcome and introduction by Jacob Kofoed (Chair of the Examination Board). The meeting aims to get a good understanding of EQE 2023 so that tutors can help candidates passing the exam next years. The committees will go through papers and questions as submitted in advance and as asked during the meeting. Jakob expressed his thanks to the committees and members of the Examination Board to make time available for this meeting.

Jakob briefly described the history of the EQE. The EQE was, for long, a paper exam in exam halls. The EQE 2020 had to be cancelled due to the pandemic that had just broken out. The EQE organization had to find a solution to allow EQE 2021 to proceed and managed to arrange the EQE 2021 as an online exam - with many more sitters than usual. Jakob indicated that that first online exam did have some technical issues, but that it went relatively well. In 2022, the exam again had the normal number of sitters and there were still some technical issues, but much less. In 2023, the exam had again more or les the normal number of sitters, i.e., about 1000 per main exam paper. Jakob presented the number of sitters per paper and the pass rates; we refer to the table above. Jakob indicated that the pass rates for Pre-Exam, A, B and C were usual, but that D was less successfully passed compared to these other papers (and, when compared to the D 2022 and 2021, but see comments to the table above and the discussion of the D paper below).

Complaints

Jabob discussed the filing and handling of complaints. If candidates want to file a complaint about the conduct of the exam, they could do so when the exam was in exam halls immediately after the exam - at that time, hardly any complaints were filed. Now, with the online exam, complaints must be filed in electronic form using the dedicated form o the EQE website, on day of the exam. Very many were filed in 2021, when candidates using the new technology ran into hickups and about 1500 complaints were filed; these were so many that they could only be answered in more general form. In 2022 less complaints were filed than in 2021, partially because less candidates than in 2021. Now, in 2023, 239 complaints were filed (Pre-Exam: 39; A: 26; B: 20; C: 45; D: 109), most relating to technical issues. All complaints go to the Examination Board. For EQE 2023, the Examination Board looked into each individual complaints in detail, and responded individually; this process ran relatively smoothly. It is believed that with more 2000 candidates these numbers of complaints are acceptable.

Misconduct

Jakob further addressed misconduct. When the exams were in exam centers, candidates had to travel to Munich, Copenhagen, Paris, …, take hotels and bring all their material to the exam center. There were tight controls around registration and around unscheduled breaks, preventing any possible misconduct. Now people take the online exam at home or elsewhere alone, with a camera showing the candidate from frontal view. The code of conduct regulations, published on the EQE website, comprise details rules on, e.g., showing ID at beginning, no head sets or ear plugs, no cell phones near, not leaving scheme unauthorized, … However, the invigilators and Examination Board have seen some deterioration about respecting the rules, e.g., people wearing headsets and some even refused to take them off/ ignored instruction to take them off. The latter is not acceptable, and the Examination Board had to take them more serious in 2023, also in view as not in pandemic crisis anymore. In EQE 2023, 36 (Pre-Exam: 17; main exam: 19) cases with a breach of rules were detected. All 36 candidates were contacted, giving facts and asking for explanation. Not all replied or convincingly explained their behaviour and some disciplinary action had to be taken, in particular deduction of points, sometimes leading from PASS into COMP FAIL or FAIL. In 2024, the Examination Board will be more strict on the rules for conduct.

Appeals

Jakob informed us that 27 appeals were received on EQE 2023. This is a relatively low number, as is is usually 40-50 (and in 2021, with almost double number of sitters and first online EQE: 150). Possibly the number is relatively low now due to the way the complaints were handled. For about half of the appeals, interlocutory revision was granted by the Examination Board in view of obvbious mistakes during marking. The other half, where the Examination Board did not consider (main) request allowable, went to Disciplinary Board of Appeal (and, as far as is know, have not yet been decided by the date on which this report was finalized in November).

Jakob concludes that the electronic exam now runs smoothly and will also be used for the future.

Outlook to 2024 [comments made spread out over the meeting by various people; information from EQE website]

The “ Information on the schedule for the EQE 2024 examination papers“ dated 27 July 2023 is alrady available on the EQE website. EQE 2024 candidates already received WISEflow access to the Compendium in September 2023.

The Unitary Patent and the Unified Patent Court have entered into force on 1 June 2023. The core EU regulations have been published in the OJ EPO in 2013 Regulation (EU) No 1257/2012 of the European Parliament and of the Council of 17 December 2012 implementing enhanced cooperation in the area of the creation of unitary patent protection (also in OJ EPO 2013, 111) – which creates a “European patent with unitary effect” (“Unitary Patent”); Council Regulation (EU) No 1260/2012 of 17 December 2012 implementing enhanced cooperation in the area of the creation of unitary patent protection with regard to the applicable translation arrangements (also in OJ EPO 2013, 132). See https://www.epo.org/en/legal/up-upcAlso see the (informal) Unitary Patent Guide published by the EPO (https://new.epo.org/en/legal/guide-up). The FAQ on the EQE website provides the particular relevance of the UPR in OJ EPO 2022, A41: “To which extent the syllabus of the EQE covers the Unitary Patent and transitional measures thereto? The syllabus of the EQE includes all updates and publications as defined in Rule 22 IPREE as at the syllabus cut-off date (31 October of the year preceding the examination). This includes inter alia, OJ EPO 2022, A41, which sets out the “Implementing Regulations on Unitary Patent Protection” before the EPO.”

The notification provisions of the EPC, Rule 126(2)/127(2) EPC, are amended per 01.11.2023, abolishing the 10-day legal fiction of notification and introducing a compensation for late receipt similar as in PCT. The amended rules are thus already in force for more than four months when sitting the EQE 2024 papers. However, the current IPREE specify that the version force on 31.10.2023, just one day before being amended. In view of this, the Supervisory Board decidedDecision of the Supervisory Board of the EQE dd 26 June 2023, EQE website (link “ EQE 2024: Notification and time limit calculation“) to take the amended version as default for EQE 2024, while allowing also the earlier version:

“1. Candidates sitting the EQE 2024 (pre-examination and main examination) can use as legal basis Rules 126, 127 and 131 EPC as in force on 31 October 2023 or Rules 126, 127 and 131 EPC as in force on 1 November 2023.
2. In the main exam papers, the default Rules 126, 127 and 131 EPC to be applied are those in force on 1 November 2023. If the candidate chooses to apply Rules 126, 127 and 131 EPC as in force on 31 October 2023, this must be clearly indicated.
3. Candidate papers will be marked accordingly.”

New EQE

The EQE 2024 will still be according to the current format. Dates for EQE 2024 and EQE 2025 have been announced on the EQE websiteSee https://www.epo.org/en/learning/professional-hub/european-qualifying-examination-eqe » Dates for the EQE 2024 dated 20 September 2022, and Dates for the EQE 2025 dated 20 July 2023It seems to be envisaged that the New EQE will be introduced in a staged way, and that there will still be current A, B, C and D papers in (at least) 2025. Please monitor OJ EPO and the EQE website for information about the New EQE and its introduction and transitional measures, as well as to whether the so-called Foundation paper of the New EQE will already be introduced in 2025 and, if so, whether the (currently announced) Pre-Exam 2025 will still take place..

The New EQE was not discussed during the Tutor meeting. It is expected that more information will become available in Q1 2024.

Why so few questions from tutors this year?

A tutor commented that the number of questions submitted in advance and the number of questions asked and comments made from the participants was quite low, and hence the degree of interaction quite limited. He indicated that he believes that to be largely due to the online format of the meeting. When the meeting was still in-person, there was a lot of “formal” interaction in the sessions between tutors and committee and Board members, as well as a lot of “informal” discussion during the breaks and other social moments around the meeting. He expressed a strong wish to again organize the meeting in in-person, or hybrid, format. Other tutors expressed their support for this request.

4) Paper A by Andrew Hards and Anna Valborg Guðmundsdóttir

In 2023, Paper A was again held electronically within the traditional 4 hrs without being split into parts. Just before the examination, candidates were allowed to print the prior-art documents and the drawings of the application, but not the Client’s letter.

Paper A was about a device for treating skin problems, such as healing wounds, wherein the device is worn on the skin. The invention related to the generation of an electric field by a triboelectric effect, which is driven by movements of the body.

D1 described an electric plaster for accelerating the healing of a wound by applying an electric field to the wounded skin, however, using a battery as power source. D2 disclosed a sensor of muscular activity, which works based on the triboelectric effect, to generate electricity by body movement but not for applying it to the skin.

The invention lies in substituting the wound healing device battery with a triboelectric power source such as known from D2.



The main device claim had to be formulated with functional features and could provide up to 40 marks. The cosmetic use claim may have compensated the chemical candidates in this mechanically orientated exam somewhat but that only garnered 10 marks.

A particular challenge was to cover all embodiments by one independent claim and to avoid unnecessary limitations, in particular in view of the type of wearable device, e.g. plaster or bandage, and the materials of the devices.

The challenge for the use or method claim was to limit the claim to a cosmetic use so as to avoid the restriction of Art. 53(c) EPC and thus excluded any therapeutic use. It was necessary to explicitly exclude the therapeutic healing of wounds, i.e. a formulation of the type “wherein the device is worn on intact wrinkled skin” or the like had to be included to receive full marks. The use claim should also make reference to the device claims.

For the drafting of dependent claims, up to 35 marks were obtainable. These dependent claims should cover the different embodiments shown in Figs. 2 to 4 and the different material options, in particular those which have been disclosed in the client’s letter as being advantageous. Further dependent claims were expected to be directed to interdigitated electrodes and the presence of an antibacterial composition. The examination committee also expected a dependent use claim, which was directed to the combination advantage of the use of an interdigitated shape of the electrodes (Fig. 5) on intact skin.

For the introductory part of the description, including the citation of prior art, an indication of the problem to be solved, and an explanation of how the invention solves the technical problem, a maximum of 15 marks could be obtained. Up to 5 marks were received for the proper citation of the prior art and the explanation of the relevant features. The formulation of the technical problem could be awarded with up to 4 marks, but only when D1 was used as a starting point. A problem formulated over D2 was less pertinent and thus gained only 2 marks. Another 6 marks were available for a discussion of the solution to the problem provided by the invention, however, the solution had to be consistent with the independent claim.

Overall, the examination committee considered the paper to be balanced and cover broad aspects. However, the examination committee was also worried that the topic of paper A was slightly too mechanical, so that chemical candidates might have had a disadvantage in solving the paper. The paper focussed on formulating appropriate functional features, but this is in any case a valuable skill regardless of the technological background of the candidates. The pass rate of the paper was at about 70 %, which is relatively high, but in line with the recent Paper A pass rates, which have ranged between 60 and 80 %.

5) Paper B by Luis Ferrera & Harrie Marsman

For Paper B, Wim van der Poel (EPO, Coordinator Examination Committee I and member of the Examination Board) informed the attendees that he was the main author for Paper B this year. He thanked not only co-drafters Liz Elmhirst and Sami Aromaa (both epi), but also made clear that there was a considerable number of persons involved in finalizing an examination paper.

Wim informed that the paper had a pass rate of 61.52%, complemented by a compensable fail rate of 9.17%. Wim noted that the Committee views with a positive light a pass rate around 62%. In addition and triggered by a question, Wim emphasized that it is always a goal to draft a paper with a mixture of mechanical and chemical aspects, which should be accessible to candidates both with a mechanical or a chemical background.

Paper B 2023 was inspired on an acquaintance of Wim which is suffering from diabetes and has to regularly check blood glucose levels. This check requires a droplet of blood and a clear improvement would be to strive to requiring smaller blood droplets.

While reading into the technology of the apparatus used, Wim saw good possibilities to have a paper dealing with selection inventions for sub-ranges, noting that the Guidelines changed recently on this subject. Instead of three requirements for having novelty for selection inventions, the Guidelines nowadays only require two requirements, viz. a narrow sub-range and the selected sub-range being sufficiently far removed from any specific examples in the prior art. The third requirement relating to the selection being purposive was removed from the Guidelines, because of developments in the case law.

On a question as to what was done with candidates who used the Case Law Book that is not yet in line with the amended Guidelines, it was noted that most, if not all, candidates worked with the Guidelines. And, this was good, because there was no real argument to be used to show a purposive selection.

Just like for the presentation on Paper A, the presentation on Paper B closely followed the Examiners’ Report that, according to our humble opinion, was detailed and of good quality.

Wim gave an overview of the paper, shortly discussed the application as filed and the problems underlying it, the prior art, the official communication, and the client’s instructions and claims.

There were two prior art documents, one of them being only a document citable under Art 54(3) EPC. This document, D2, should not be used in the inventive step discussion, and it was considered a serious mistake to do so. Some candidates that used D2 as secondary document lost marks. It was also mentioned that candidates that used D2 in inventive step were already seemingly struggling with the overall paper; the Art 54(3) citation was not seen, in itself, to have been a significant stumbling block for candidates. On average, the inventive step argumentation given by the candidates for this year’s exam was quite good.

In the Communication, the Examiner indicated that D2 was novelty destroying for all five claims as filed. This point was apparently accepted by the vast majority of candidates and is in line with the usual practice that in the EQE paper B the Examiner’s position is correct.

One of the tutors observed that an argument that D2, read in combination with D1, did not directly and unambiguously disclose the subject-matter of claims 2 and 4. From the Examiners’ Report it is unclear how such an alternative solution would have been marked. Wim stated that such candidates would probably have received some marks, if arguing convincingly. However, such cases were not reported to him.

In respect of a question regarding computer implemented inventions (CII), it was confirmed that a basic knowledge is expected from all candidates, but not at the same level as practitioners in the field (for example, a complete paper based on CII would, in all likelihood, be too selective).

The presentation also included a mention to the two different effects provided by the two independent claims (smaller amount of blood can be used; more accurate measurement) and corresponding objective technical problem to be solved. As expected, the second inventive argument provided fewer marks than the first.

In the communication, it was also noted that there was an error in claim 3 or in paragraph [017]. The units ml and µl were used for indicating the same property. The candidates were expected to request for a correction citing Rule 139 EPC and using the document supplied by the client. This request for correction should preferably be supported by additional information, such as a reference to another document, in this case D1.

Also, an objection under Rule 43(2) EPC regarding claims 1 and 5, directed to a strip and a measuring device, respectively, as originally filed had to be dealt with. This could be easily dealt with while referring to the exception of the plug-and-socket type in Rule 43(2)(a) EPC by adding the corresponding feature of the array detector. Candidates who solved this Rule 43(2) EPC objection by introducing a claim directed to the use of the strip together with an array detector lost marks.

The applicant’s letter raised the issue of wishing to also incorporate hydrophobic membranes in the device. This wish could not be met. There is simply no support to broaden from hydrophilic membranes and violating this resulted in a considerable loss of marks.

As was the case with the previous single Papers B, the points for the claims are coupled to the amendments of the set of claims as suggested by the client.

As indicated above, the Examiners’ Report is quite detailed and should be read attentively. The paper was well appreciated by the participants to the EQE tutors meeting.

6) Paper C by Sander van Rijswou

Presentation at the annual meeting of EQE tutors and members of the EQE examination committee. The presentation was by Pelayo Fernandez Plaza but presented by Sophie Creux. Questions were answered by Sophie Creux, Paolo Provvisionato, and Celia Martinez Rico.

The presentation closely followed the Exam report for Paper C, with relevant excerpts from the report on the slides. These minutes focus on comments that seem to add to the Exam report.

Effective dates of the claims

The priority application was filed by two applicants, whereas the patent application was filed by only one of them. No transfer of priority right had taken place.

Following the EPO practice, reflected in the Guidelines of 2022 and 2023, A-III, 6.1, the priority is not validly claimed. Reference was made to the recent decision from the enlarged board of appeal about priority [G1/22, G2/22]. Nevertheless, candidates should apply the law as it was at the time.

List of Evidence

Annex 2 and Annex 5 required more discussion.

A2 itself was published after the filing date of A1. But it’s evidence that a public prior use took place before the filing date. It should be noted that the internal structure of the pedal was also made available to the public through posters which were displayed.

A5 is a screenshot of a social media post. Following EPO practice, the date stamps of the post and the comments are considered to be reliable publication dates. Note that A5 has two dates.

Claim 1 - Lack of novelty (A5) and discussion on other attacks

Novelty A5

This was the only expected attack. It should be noted that road racing implies limitations; not all prior art pedals were suitable for road racing. The sensor was also an important point: a pedal angle sensor is enough, see A1[6].

Novelty A4

Prior to the meeting, a tutor submitted that he considered a novelty attack based on A4 also a valid attack. This attack was not expected, and considered incorrect because A4 does not provide information about where the dead spots are in the pedal stroke.

Although A1[6] mentions that there could be other methods for measuring dead spots, none are specified. A4 discloses strain gauges, which can be used to measure power. They are mentioned in A3, A5, and A7. However, they measure total power or power balance between left and right. But what is needed is a sensor that measures power in a stroke. A strain gauge is not enough; you need information that tells you where you are in the stroke like an angle detection. Gauge sensors, without any further indication, are not suitable for detecting the dead spots in the pedal stroke.

The time invested in a novelty attack based on A4 was not lost however, as the analysis of A4 is useful for attacking claim 2. There are thus few marks lost because of this. Missing the novelty attack based on A5 was penalized by the corresponding number of marks.

Inventive step using A7

In passing, a brief mention was made about a possible inventive step attack on Claim 1 using document A7. This attack was not expected but may have received points based on the merit of the attack. The other document(s) used in this attack were not mentioned. [Perhaps this may have been inventive step A4+A7, since A7 establishes a ‘power profile along the stroke’ thus overcoming the shortcoming of A4, SMR]

Claim 2 - Lack of inventive step (A4 + A2)

For the inventive step attack for claim 2, it is needed to change to A4 as the closest prior art. A5 is not the closest prior art because A5 has a very different structure, and so is not suitable to start from. Also, document A2 is not the closest prior art, because it discloses a stationary bicycle pedal.

Claim 3 - Lack of inventive step (A4 + A2 + A3)

A4 is still a good starting point, and it additionally has a bicycle computer. A2 is even further from claim three because there is no bicycle computer, only a PC workstation.

Claim 4 - Lack of novelty (A7)

A novelty attack based on A7 was the only expected attack. A7 is the only document disclosing a bicycle with a chain drive. The attack required an explanation of what a chain drive is.

Claim 5 - Lack of inventive step (A7 + A6)

A6 has preferred and non-preferred embodiments. An explanation was expected for why these specific ranges would be combined.

Claim 6 - Added subject-matter

There is added subject matter because ceramic ball bearings are isolated from its context in A1. To achieve a reduced friction and an increased life span when compared to standard steel bearing, the Zirconia ball bearings and ceramic races need to be combined. There is no information that ceramic ball bearing could be extracted.

Claim 7 - Lack of inventive step (A3)

Claim 7 comprises a mix of technical and non-technical features. It is expected that candidates apply the Comvik problem-solution approach of Guidelines G-vii, 5.4.

Note that in Paper C of 2022 there was also a mixed type invention, for which it was expected to apply the Comvik approach.

According to the Guidelines first one should state what the technical features are (a computer and displaying pedaling efficiency being at 80% or below), and what the non-technical features are (a heart or an angry icon).

The closest prior art is then selected with a focus on the features contributing to the technical character.

The bicycle computer of claim 7 needs to be suitable for the system of claim 4. This means only that the bicycle computer needs to be able to communicate with the sensor of that system.

A3 is considered to be the prior art because it discloses a bicycle computer, which is suitable for communicating with a sensor and which already displays efficiency information.

Although, A7 discloses a bicycle computer it is not the closest prior art because it lacks a display for pedaling efficiency.

For A3, the only distinguishing feature is that the display of whether the efficiency surpasses or is below 80% is a heart icon or an angry icon. This feature does not make a technical contribution because it depends on the user’s preference.

Distribution of marks

  • Effective dates of the claims and prior art (10 marks)
  • Claim 1 (11 marks)
  • Claim 2 (18 marks)
  • Claim 3 (15 marks)
  • Claim 4 (9 marks)
  • Claim 5 (17 marks)
  • Claim 6 (6 marks)
  • Claim 7 (14 marks)

Questions from tutors

Question: Although the marks appear to be balanced between the two parts, the effort needed by the candidates was not balanced, as many candidates appeared to run out of time in Part 1, but they had sufficient time for Part 2. Do you have any comment on that?

A split paper allows some rest during this online exam. It would be demanding to sit the whole paper online. This comes with the drawback that candidates are not allowed to answer the first part of the paper, during the second part. If this is done, zero marks are awarded.

This year one of the annexes was only provided in part 2, because candidates have to spend more time in the first part. Nevertheless, reading material that is not useful is part of the challenge.

Question: What is the situation in paper C: do we also need novelty or inventive step attacks in paper C if a claim violates Art.123(2) EPC?

It depends on the paper. This year, there is a clear variation of Article 123(2), and therefore that is enough to kill the claim. It is not a rule that added subject matter is sufficient, though until now this has always been the case. In future there could be a paper in which the added subject matter situation is not so clear. One of the tutors remarked that some older papers did have auxiliary attacks on added subject matter claims.

Finally, a reminder was given that candidates should read the instructions to sit the exam and know how to behave during the examination. There have been incidents with people wearing headsets, having cell phones, and leaving unauthorized. Next year there will be consequences for not adhering to the rules.

7) Paper D by Roel van Woudenberg and Zsofia Pintz

The D committee was represented by Tiem Reijns (epi, chairman D, member Examination Board), Josef Schriefl (EPO, D1) and Simone Fausti (EPO, D2).

D1-part: summary of the paper

This year’s D1 came in two parts: 1 first part of 1 h 35 min and 27 marks (D1.1) and a second part of 1 h 40 min minutes and 28 marks (D1.2), i.e., a total of 55 marks. The D1 had a well-balanced mixture of EPC and PCT questions, with common topics as well as some less-familiar topics. Candidates that were well-prepared, with a sound legal knowledge and familiar with their legal reference books and other material should have been able to score 50-60% or more out of the 45 marks within the time available.

Some topics were common topics that candidates could have expected, such as dealing with lack of unity and divisional applications. Some topics were not so familiar, e.g. the excuse procedure in PCT national phase.

The online access to the Guidelines GL/EPO and GL/PCT-EPO in html-form (only) was convenient and may have been of additional help.

The first D1 part, D1.1, consisted of three questions of 10, 10 and 7 marks, i.e. a total of 27 marks, for which 1 h 35 minutes were available, followed by a break.

In Q.1, two pending divisional applications had to be analysed for patentability, both of which were filed out of EP-A. EP-A disclosed two inventions, the first invention comprising two alternative solutions. EP-A was a recently refused application which was still pending until the expiry of the period for filing a notice of appeal, so a new divisional could still be filed of it for the second invention. Candidates were expected to discuss the removal of an optional feature in one of the divisional applications and that the omission of a feature that is indispensable for the function of the invention was not possible in the other divisional application.

Q.2 related to an international patent application, where the priority claim included a typographical error in one digit of the application number of US-B. The applicant received an invitation from the EPO as receiving Office to correct the priority, but this was overlooked and the invitation was not replied to. Candidates had to discuss whether it was still possible to rectify the priority claim in the international phase and calculate the last day to file a demand for international preliminary examination. Candidates were expected to rectify the priority claim under Rule 91.1 PCT, and to indicate the necessary steps for requesting the rectification.

The topic of Q.3 was a recently granted European patent, a revocation request filed for the same patent and a debit order for said revocation filed via fax. The question also mentioned that the renewal fees were not paid in Germany and that another company is concerned about infringing the patent, also in Germany. Candidates were expected to analyse the current status of the revocation proceedings and discuss why the company concerned about infringing the patent should file an opposition. It had to be realised that the purpose and effect of an actively filed opposition (ex tunc) is different than the lapse of the patent (ex nunc).

The second D1 part, D1.2, consisted of another three questions of 7, 11 and 10 marks, i.e., a total of 28 marks, for which 1 h 40 minutes were available.

Q.4 related to a recently revoked patent. A notice of appeal was already filed against the decision of the opposition division; however, the notice of appeal was erroneously filed in the name of a different company, i.e. the previous patent proprietor. Candidates had to describe the current status regarding the admissibility of the appeal and discuss how the situation may be improved. It was expected to conclude that according to the current situation, the appeal was likely to be rejected as inadmissible. However, there were at least two possible ways for a successful correction (G 1/12); either under Rule 139 EPC or Rule 101(2) EPC.

Q.5, the question for the most marks in D1 (11 marks), had a PCT related topic, in particular a missing abstract in an international patent application. As the application was declared withdrawn, candidates were required to analyse the legal consequence of said withdrawal in the designated states if nothing is done; and then discuss if and how applicant could revive the application before the EPO. Candidates were expected to state that the application ceased to have the effect of a regular national application in each designated state and then indicate the steps of the excuse procedure to successfully revive the application before the EPO.

In Q.6, there was a pending opposition against a patent, which was granted with a single claim to product P. The patent described the product P, a method only resulting in product P, and the use of product P as a fertiliser to enhance plant growth. The opposition included a document under Article 54(3) EPC as a possible novelty destroying prior right, disclosing the same product P, wherein P is obtained by a different method, and the use of product P as a detergent. Candidates had to discuss how the patent should be amended to provide the maximum scope of protection; while, of course, also meeting the requirements of Art.123 and 54 EPC. In the second part of the question, wherein the patent described product P for use as a medicament rather than describing the use of product P as a fertiliser, candidates were expected to realise that the use claim is excluded from patentability and had to be reformulated as a purpose-related product claim under Art. 54(4) EPC.

The full answers to the questions are given in the Examiners’ Report. It also indicated, as every year, important guidance for answering (e.g., “Candidates are reminded that they should pay attention to the way the questions are asked”) - most of these points were also emphasized at the meeting (see above under “General remarks” and “Answering and marking”).

D2-part: summary of the paper

This year’s D2 was a 45-mark for which 2 h 45 min was available, the shortest D2 exam so far.

The paper was of a quite common design, with quite some different subject-matter. A careful patentability assessment was needed, followed by a who-is-free-to-do-what analysis, and then an improvement question. As always, it was recommended that candidates follow the scheme for the questions in their response. In this case, to discuss the patent situation by subject matter, as asked in the exam. This way of structuring the answer intended to help candidates not to lose marks because of an incomplete analysis.

There were no strange legal topics, but the length of the paper was actually longer than in previous years and last year (in 2023: 1479 words in the exam in 2 h 45 min vs. in 2022: 1408 words in the exam in 3 h 20 min).

The subject of the paper was nappies with control unit, electrodes, warning means, substances; electrodes for detecting presence of urine, special electrodes G or P for measuring glucose or pH and electrodes made from metal, silver, copper, platinum, gold, or conductive plastic. 

Candidates had to deal with a competitor with a recently granted patent with quite a wide scope, validity of priority, the consequences of the invalid priority, undisclosed disclaimer, and a presumed Art.54(3) that turned out to be a Art.54(2) prior art. Other legal issues were added matter, non-unity, non-claimed subject-matter, and a R.71(3) communication already being approved. The client was based in Sweden, but producing and selling in EP, the US and China. However, the client only had EP applications pending, so a PCT application had to be filed for the nappies comprising substance X, because the International application can provide protection in US and China.

According to the Examiners’ Report, the candidates are expected to analyse the available prior art given in the paper, to discuss the effective date of claimed subject-matter, and to conclude on patentability and/or validity of the claim(s), even -as more often in D2- if it is stated in the exam paper that a search report only contains documents of category A, or that a communication for the intention to grant has been issued. Some candidates did not provide a complete analysis as outlined above and were not awarded full marks.

As in 2021 and 2022, the paper had to be taken fully from the screen in Wiseflow - nothing was printable. The pdf of the paper could be viewed side-by-side with the editor, without an annotation possibility; or in one or more separate tabs, with limited annotation possibility. No calendars were given with the exam paper, but candidates had to bring their own lists of Saturdays and Sundays and had to check EPO closure dates in the OJ.

General remarks (Tiem Reijns)

Paper D 2023 had a pass rate of 38,3% and 10,35% compensable fail. This is a relatively normal pass rate for paper D from before Covid (pass rates 33-53%, compensable fail 8-13%), after a few high years of artificially high pass rate due to significant neutralizations in 2021 (one complete part of 25 marks, due to technical issue at start of D1-1) and 2022 (one question, 5 marks, due to date error in English version).

The Committee was actually expecting a higher pass rate, as there were only few resitters due to the higher pass rates in 2021 and 2022 and also because candidates got used to the online platform, the paper was quite straightforward without any substantial surprises in the content of the paper, most candidates answered all questions, and because the online materials helped candidates in finding the legal basis more easily [note from the editor: the latter may not be true for all candidates, as some may consider printed reference books to be more efficient and easy to use than online resources, especially when those books are annotated with personal comment].

In the 2023 paper however, the legal questions scored lower than expected. This is largely due to a continuing trend of citing less and less legal basis; apart from Articles and Rules, the committee sees little cited legal basis (case law, Guidelines and OJ EPO are generally ignored. The question about opposition and revocation (Q.3) included a debit order was scored disappointingly low; this was unexpected as debit order questions are normally quite well-scored. The legal assessment (part 2) was considered straightforward and candidates scored according to expectation. Few candidates discussed non-EPC markets.

Pre-received questions relating to general aspects (Tiem Reijns)

Unscheduled breaks
A tutor noted that, currently, unscheduled breaks are not allowed in the D1 parts. However, the D1 parts have grown to be as long as 1h 40 minutes for 28 marks, and may even become as long as 2 hours if a 60-mark D1 would have a 33 mark and a 27 mark part. With D1’s getting as long as 1 h 40 min in 2023 and even 1 h 45 in 2024, should one not also allow unscheduled breaks in such D1 parts? (What is the limit: 1h30, 1h45, 2h, 2h30, 3h?)

Answer: D1 can indeed be more than 40 marks, as floating between 40 and 60. As a limit for unscheduled breaks, a 2 hour cut-off was set. Thus, unscheduled breaks are given for A, B, C and D2, but only after 60 min and no longer than 3 min. As D1 will not be longer than 2 hours, D1 can do without.

The Examination Board was requested by the tutor to allow unscheduled breaks in paper parts of any length, also during Pre-Exam parts and D1 parts.

Tiem answered that for security reasons, unscheduled breaks are only given in accordance with the instructions to the candidate. exceptions may be made for e.g. medical reasons if requested in advance.

Length and number of D1 questions
A tutor observed that this year’s D1 part has 6 questions of 10, 10, 7, 7, 11 and 10 marks, no short questions, and the average above 9 marks. The document “Information on the schedule for the EQE 2024 examination papers” shows a first 26-mark part of only two questions, such that there will be a question of even at least 13, but possibly 14-16 marks or more. Before 2020, D1 papers used to have 5 or 6 questions for 40 marks, with the number of marks per question between 4 and 10, and mostly between 6 and 8, with the average around 7.

The tutor wondered whether having only a few (in D1 2024 even only 5) high-value questions may make failing D due to misunderstanding of a single legal principle more likely, and unwanted. He also wondered whether it was in line with the intent of Rule 26(2) IPREE which indicates that “It [Paper D] shall comprise questions relating to different areas of the candidates’ legal knowledge” (emphasis added), from which a candidate can legitimately expect that he will be tested on a wide variety of topics and that none of the topics will have a very high weight.

He requested the Committee to comment on their choice to provide questions with such a high number of marks and such a high relative weight.

Tiem commented that the Committee has the impression that candidates to better on questions with more marks. Possible reasons: lower overall number of words and this more time to answer. D1 2024 will have a first 26-mark part of only two questions, such that there will be a question of at least 13, but possibly 14-16 marks or more.

Tiem sees no deviation from the past, as there have always been 5 or 6 questions. Thus, the number of questions is stable, but number of marks per questions varies.

The Committee does not wish to limit the length of an individual question to 10 marks, not the average in the (previously common) range of 7-8 marks, as was requested by the tutor.

Length D2 part and possible solution

The same tutor noted that, when comparing the answer to that of the last years, the D2 2023 paper as well as its Possible solution, are considerably longer than the 2021 and 2022 papers, even though only 45 marks were available, i.e., significantly less than in 2021 and 2022, and also considerable longer than the papers before 2020.

The absolute lengths (in number of words) are given below, as well as the length scaled to 60 marks in brackets:

  • D2 2023, 45 (60) marks, 2 h 45, paper: 1479 (1972) words; possible solution: 1395 (1860) words
  • D2 2022, 55 (60) marks, 3 h 20, paper: 1408 (1536) words; possible solution: 1251 (1365) words
  • D2 2021, 50 (60) marks, 3 h 00, paper: 1289 (1547) words; possible solution: 1186 (1423) words
  • D2 2019, 60 marks, 3 h 00, paper: 1183 (1183) words; possible solution: 1438 (1438) words
  • D2 2017, 60 marks, 3 h 00, paper: 1335 words; possible solution: 1303 words
  • D2 2013, 60 marks, 3 h 00, paper: 1435 words; possible solution: 1244 words

Also when not counting the number of words, but the number of arguments or sentences in the Possible Solution, this paper and its answer was by far the longest. The tutor considered that it may assumed that this length is one of the factors causing a low pass rate for D 2023.

The tutor asked what the reason is that this paper was so long and required such a long answer, and how the D Committee decides and validates whether the length of the paper is adequate for the time (and marks) available?

Tiem commented that, indeed, the number of words appears to go up, but the committee does not consider it a worrying trend: 1) the number of words in paper D varies from year to year; 2) the possible solution is drafted to help candidates prepare, not because the answers style as such is required; using full wording is giving more clarity to the possible solution; 3) there was quite some repetition to precisely define the subject-matter (“…nappies comprising acoustic warning means, a control unit and electrodes made of metal for detecting the presence of urine“); repeating this multiple times is already many words – considering the repetition, the effective length is the same as in the past; in the digital format, allowing copy-paste, these extra words are only added to the reading, not the writing (note from the editor: the paper also provided for abbreviations, so that writing could also be limited by using the abbreviations only).

Further, the main difference with previous years is the number of words in the detailed questions 1-3 of the D2 part. These detailed questions are provided to help candidates by giving guidance and they provide repeated subject-matter definitions. E.g., D2 2023 used 255 words for Q.1-3, whereas D2 2017 (metal beams) used only 71 for Q.1-3. Tiem showed a graph to illustrate that, when excluding the word count for the explicit questions from the total number of words, the net effect is small.

As to deciding on the length of the paper, Examination Committee III determined whether the part tests candidates on being “fit to practice” and does not steer on the number of words in the paper or the solution. However, the Committee aims to be concise, in D1 and D2.

D1 - Josef Schriefl (EPO)

Josef addressed the questions submitted prior to the meeting in detail.

Guidelines as legal basis

In view of the Examiner’s Report’s indication that “The examiners note that this issue has become more acute in recent years, with the 2023 answers showing an all-time low in the citation of legal basis from e.g. the guidelines and case law.” (emphasis added), a tutor asked whether the Committee can give guidance when a Guidelines citation is necessary.

Josef answered that a reference to the Guidelines is expected if an element of the answer cannot be directly derived from the Article or Rule. Corresponding considerations apply to references to case law and other provisions comprised in the syllabus.

As an example, D1, question 1s, states: “it is directly and unambiguously apparent that feature B is indispensable for the function of the first invention”. Art. 123(2) EPC: application may not be amended in such a way that it contains subject-matter which extends beyond the content of the application as filed. G 1/05: (1) divisionals not complying with Art. 76(1) at their filing date may be amended later; (2) even if parent no longer pending; (3) same principle for examining Art. 123(2) and 76(1) EPC. Guidelines H-V, 3.1: removing a feature that is indispensable for the function if the invention contravenes Art. 123(2) EPC. Thus, the reference to the Guidelines is required as that provides the basis that removal of the indispensable feature leads to added matter.

Divisional in appeal period

A tutor commented that the Possible Solution does not explicitly include that the divisional may be filed in the appeal period “also if no appeal is ultimately filed”, and asked whether the latter was necessary for full marks and whether there would be a loss of marks if a candidate suggested that an appeal was necessary for the pendency status.

Josef indicated that marks were also awarded to candidates that advised to file an appeal and file the divisional during the appeal proceedings, but that that option is less-preferred as it requires additional acts and fees - the only good reason would be to gain time, but there is no such hint in the question, nor is there a hint that the appeal could be successful. So, only those that filed the divisional without filing an appeal scored full marks.

D2 - Simone Fausti

Simone also commented on the length of the paper and the possible solution. His comments are incorporated in the General section above.

Level of detail when arguing novelty and inventive step

A tutor commented that in the argumentation of novelty, earlier Examiner’s Reports usually required a thorough, complete analysis when assessing novelty: they required to explicitly indicate each prior art disclosure, then argue why the claim was novel by explicitly identifying the distinguishing feature(s) compared to each individual piece of prior art one-by-one, and then conclude. However, the D2 2023 model solution used arguments like “The claimed subject-matter is novel, because there is no disclosure of nappies comprising acoustic warning means in the prior art”, i.e., without explicit indication of the prior art and without explicit indication of the distinguishing feature (but rather just citing the complete claim).

Simone disagreed. He argued that the possible solution of this year (and last year) does not deviate from general good practice. E.g., “The claimed subject-matter is novel, because there is no disclosure of nappies with the special electrodes G configured to measure glucose in the prior art” as given in the Possible Solution clearly mentions distinguishing feature vs an argument that would not give any identification such as “… because there is no disclosure of the claimed subject-matter in the prior art“.

Where it relates to discussing individual prior art, the D2 allows to provide information and provide analysis in selective manner; this may be in aggregated manner if situation is appropriate. An extensive discussion gets same marks as the aggregate discussion in the possible solution. No bonus for unnecessary words. Level of detail in discussing novelty and inv step depends on the situation in paper D; not as in ABC, where the way to discuss it is tested.

Simone commented that marking is candidate-friendly: looking for proof that candidates relies on the relevant effect when arguing inventive step; wording used not relevant. In an inventive step discussion, a general reference to an unspecified improvement is not sufficient; for marks, specifying the improvement/ surprising effect is necessary, or an argument like “only known from remote field” in support of inventive step.

Art. 123(2) and inventive step grounds of opposition

A tutor commented that the Possible Solution correctly provides in Q.1-a that: “By claiming electrodes not made of silver, there is an undisclosed disclaimer in the granted claim of EP-Z, because silver was not mentioned in EP-Z. An undisclosed disclaimer can be allowed to restore novelty over an accidental disclosure and/or a prior right. EP-Y is […]. Hence, the undisclosed disclaimer is not allowable, as it represents added matter. Altogether, the claim of EP-Z is invalid.” and in Q.3 “Based on the analysis made in the answer of Q1, file a notice of opposition on the ground[s] of […] added matter.”

However, the Possible solution also provides in Q-1-a that “The effective date is therefore the filing date of EP-Z.” and “The subject-matter of the amended claim is novel because none of the prior art documents discloses electrodes, which are made of metal other than silver. However, it does not involve an inventive step over the handbook, because the distinguishing feature is a well-known alternative for nappy electrodes to detect the presence of urine.” and in Q.3 “and in Q.3 “Based on the analysis made in the answer of Q1, file a notice of opposition on the ground[s] of inventive step […]”. And, as to the latter, the comments to Q.3 indicated: “Fewer proposed the ground of inventive step.”

The tutor was surprised that it seems that the inventive step attack was needed, in addition to the 123(2) attack, for full marks, as a 123(2)-violating claim cannot be given an effective date (so would not be legally sound; even though in real life one would always add one…), and the 123(2) attack alone would be successful. He asked the committee to comment.

The tutor also indicated that he appreciated the inventive step option in case candidates had not recognized the (relatively complicated) 123(2) problem: the possibility for an inventive step would reduce the knock-on effect as it could still allow those candidates to conclude that the claim with the disclaimer was not valid (as not inventive in that case) and would still allow a successful opposition (albeit in the wrong ground). Reducing such knock-on effects is good and consistent practice in D2 papers since many years, and much appreciated.

Simone answered that “fit for practice” supports to also require raising inventive step as a ground besides 123(2), and he also referred to the statement of the tutor himself that he would always file that in real life. It is an additional ground and a fallback position. However, a candidate would hardly loose marks when not including inventive step as a ground of opposition; those marks are negligible compared to the total number of marks for EP-Z assessment and opposition. The main purpose was indeed to reduce the knock-on effect.

8) Pre-Exam by Nico Cordes and Roel van Woudenberg

Examination Committee IV (Pre-Exam) was represented by Stefan Götsch (EPO), Volker Franz (EPO) and Joanna Moore (epi).

We will first give some general comments on the paper, then discuss the legal part of the paper, followed by the claims analysis part, and then discuss the session at the Tutor meeting.

General remarks

This was the third online EQE with the Pre-Exam being held in a similar way as in 2021 and 2022: the Pre-Exam was split into 4 parts and the questions had to be taken almost fully from the screen (only the description of the application and the prior art for each of the claims analysis parts were printable). Each part had to be completed before the start of the next break, with the next set of questions only becoming available after the break in between two successive parts. The split into parts means that candidates were not free to allocate their time as they see fit across the different parts of the paper. To compensate for this restriction, the total duration of the paper was extended: for every part, an extra 10 minutes was available so that each part lasted 70 minutes. Once the time allowed for a part had elapsed, it was not possible to go back to that part. The Pre-Exam 2024 will have a similar design.

As in 2022, the order of the four statements in each of the questions was randomized In the legal as well as the claims analysis parts, i.e., it was different for different candidates. Also, the order of the questions was randomized in the legal parts (e.g., the sequence used below is the one shown in the Compendium), but not in the claims analysis parts.

All questions, statements, prior art documents and claim sets were provided in all three official EPO languages. As in Pre-Exam 2022, candidates could view the question body in a single language of choice in a first column on the left half of the WISEflow window, while the statements were presented in a second column on the right half of the window in all three languages without the possibility to select just one: this resulted in quite some scrolling, especially in the claims parts, where a single webpage showed four tri-lingual statements with True/False tick boxes.

The online Pre-Exams have the same syllabus and character as the earlier paper exams, and candidates are -as before- allowed to bring any printed reference material. During the exam candidates also had access to the EPO Legal Text pages, so including the full EPC Articles and Rules, Guidelines, GL/PCT-EPO, National Law Tables, Case Law, and the Euro-PCT Guide (HTML versions), and in this year also to some of the PCT Legal Texts, in particular the eGuide of the PCT Applicant’s Guide (Introductions and Annexes) and the Article and RulesIt was indicated that candidates need to monitor the Notices on the EQE website to check what is available online in Pre-Exam 2024.. Access to the full pdf-versions of longer legal texts was generally not available. Access was to the live versions, so to the versions in force on the date of the Pre-Exam (so not the version of 31.10.2022 acc. Rules 2 and 22(1) IPREE) and answers based thereon were also acceptedFor the questions of Pre-Exam 2023, the differences between both versions of the Guidelines had no effect on the answers..

Some questions and statements were extensively debated in comments on the blogs, both the legal part and especially the claims part. At earlier occasions, the Committee and Examination Board indicated that they consider the blogs and their comments when checking whether statements may need neutralization. However, In the legal part of this year’s exam, no statements were neutralized.. In the claims part, one statement was neutralized; however, even though some candidates and tutors expected so, the “water-based” statements (see below) were not neutralized and the committee kept to their interpretation of “water-based solvent system” being “only water”.

Legal part

The legal part of this year’s Pre-Exam had similar style and a difficulty level as in 2022, 2021 and 2019, and a bit more difficult than those of 2016, 2017 and 2018. As in the last few years, the legal questions were a mix of questions addressing several topics that were to be expected (such as filing date requirements, languages) as well as several less standard topics (such as entitlement, transmittal under PCT Rule 19.4) which well-prepared candidates would have been able to find in their EPC/ PCT/ reference materials/ Guidelines, albeit with some effort. Surprisingly, representation and opposition were not tested (except for third party observations in opposition in 8.4).

Several questions were amended versions from earlier Pre-Exam questions, with names of parties, names of applications and dates adapted. Candidates that practiced the earlier Pre-Exams using the Compendium or Q&A books will have recognized so when working on the paper and will have benefitted from having used them in their preparation.

Well-prepared candidates having good knowledge of the EPC and PCT and knowing their material well (reference books, EPO Guidelines, PCT Applicants’ Guide, Euro-PCT Guide, Q&A books, flowcharts, other course material) for fast look-up should have been able to answer most of the statements correctly and well within the time available, provided they had their material updated to the legal status of 31 October 2022 (the cut-off date for EQE 2023). The EPO Guidelines in particular play a key role to get to the right answers. Candidates with a poor preparation will have found those questions considerably more difficult than the questions from earlier years.

Q.1 was an adapted version of Q.1 of Pre-Exam 2019. Unlike the usual practice of “today” being the date of the exam (17 March 2023), the question indicated as “Today, 6 March 2023”; the origin of this deviant date seems to be that the Committee wanted to keep the Rule 134(1) extension of the priority period, which was an element of the original version (Pre-Exam 2019 was held on a Monday). Q.1 tested filing aspects, in particular filing date, language of filing and priority.

In Q.2, the effect of missing drawings and late-filing of those were tested. E.g., it tested whether a candidate was aware that not responding to an invitation to file missing parts of the description or missing drawings does not result in a deemed withdrawal but in then all references to the missing parts are deemed to be deleted (Rule 56(4) and Guidelines A-II, 5.1).

Q.3 was an adapted version of Q.5 of Pre-Exam 2019 and tested recording of changes under the PCT, who can validly file a first application at the EPO under EPC and at the EPO or OB under the PCT. The change of the question led to the complication that one of the applicants now lived in France (in 2019, Werner lived in Berlin), such that national security provisions apply to any application (whereas in Germany they only apply for applications containing state secrets - AG-IP Annex B); it is unclear whether the Committee was aware that some candidate may have been confused by this and did not know (as it is nowt documented in any of the documents of the exam syllabus for Pre-Exam 2023) whether violating national security provisions (requiring filing with the national office of France) results in an invalid filing or not (note: for the purposes of the EPC or PCT, it does not have an effect of the validity of the filing).

Q.4 described an entitlement situation and tested possible follow-up actions and their effect. Candidates may generally not be very familiar with entitlement issues from their daily practice, and may thus consider this a difficult question. However, the question could be fully answered using the Guidelines (2022) A-IV, 2 and sub-sections, so that a candidate that is well familiar with the EPC and his/her reference material can nevertheless answer this answer efficiently.

Several PCT aspects were tested in Q.5, such as (no) change of language of proceedings on entry (G 4/08; Guidelines E-IX, 2.1.3), and effects of withdrawal of a priority claim on effective date of claim 2 and on the entry time limit. In this question, one statement was neutralized, because it required a calendar for 2024 to know that 31 August 2024 was a Saturday so that the time limit for EP entry expired on Monday, 2 September 2024.

Q.6 was substantially the same as Q.2 of Pre-Exam 2014 and directed to divisionals: effect of matter extending beyond the parent application on its date of filing, prior art effect, pendency and renewal fees.

Q.7 was another PCT question, addressing fees and translations for international search, and addressing the effect of filing with the French office in English (received on behalf of IB as rO, transmittal to the IB, Rule 19.4). The later topic had a similar complication as Q.4, as R.19.4(b) which provides that the transmittal to the IB is done “unless prescriptions concerning national security prevent the international application from being so transmitted”, causing some candidates to doubt what the French office would actually do in this case, as may be caused by the condition in R.19.4 that “that Such transmittal may be subjected by the national Office to the payment of a fee, for its own benefit, equal to the transmittal fee charged by that Office under Rule 14”. However, the two statements relating to R.19.4 were not neutralized.

In Q.8, aspects of third party observations were tested. Most statements could be answered based on Guidelines (2022) E-VI, 3. Candidates that used the later version of the Guidelines may have had a slight advantage as section E-VI, 3 was significantly amended.

In Q.9, thew applicant just received an invitation under Rule 70(2) and Rule 70a(2) EPC while the European search opinion stated that claim 1 lacks novelty. The question tested requirements for the reply to the invitation, time limit, requesting accelerated examination and claims fees.

Q.10 related to the designation of inventors and their rectification, waiving the right to be mentioned and who can pay fees. Candidates may have overlooked that Rule 19 EPC was amended per 1 April 2021 so that the EPO no longer inform the inventors in a communication about their designation as inventor.

Claims analysis part

The claims analysis part had a similar style as in EQE 2021 and 2022, in that the claims analysis part was structured into two parts which were separately identified as part 3 and 4 (with parts 1 and 2 being the legal parts). As in the EQE 2021 and 2022, parts 3 and 4 each concerned a separate case with their own set of questions, rather than one case of 10 questions as in previous years:

  • Part 3: Cleaning utensil (questions 11-15)
  • Part 4: Erasable inks (questions 16-20)

Each of the parts 1-4 was given their own timeslot with each timeslot being divided by a break of 20 minutes (between parts 1 and 2, and between parts 3 and 4) and a larger break of 80 minutes between the legal parts and the claims analysis parts.

Part 3 concerns a cleaning cloth which combines relatively simple mechanical aspects (an arrangement of layers) with some chemical aspects, for example relative weights expressed as ranges and different types of materials, such as sponge material, woven material and different types of polymers. Three different embodiments are described. Embodiment 1 is a basic embodiment, embodiment 2 adds a reinforcement layer, and embodiment 3 has fibres which form loops on the upper layer of the cleaning cloth. Each embodiment is accompanied by a figure showing its cross-section.

Part 3 contains only one prior art document, namely D1. The legal status of D1, being Art. 54(2) EPC, is explicitly indicated (i.e., it is not needed to determine the legal status using a comparison of dates), which is typical in the claims analysis part.

The claim set I which was initially presented included seven claims, namely one independent claim and six dependent claims. Several (sets of) other claims were introduced during later statements.

Substantive topics in part 3 included scope of protection, novelty, inventive step, and quite extensively, support for amendments. Of particular interest were:

  • Q12, in which candidates had to assume that a claim set was filed after the description and drawings in response to a communication under Rule 58 EPC and had to determine whether individual claims met the requirements of Art. 123(2) EPC. This question demonstrated the peril of filing claims as amendments under Rule 58 EPC.

  • Q14, which contained a statement about the formulation of the objective technical problem, however without indication of which document to consider as the closest prior art. The statement was not neutralized by the Examination Board. Fortunately, it appeared that candidates did not particularly struggle with this statement, perhaps given the fact that only one prior art document (D1) was available in this claims analysis part.

Part 4 dealt with the topic of erasable inks.

Notable aspects of part 4 included the following: the application was a PCT application of which the filing date was indicated to be 16.08.2019.

The prior art document D1 was an Advertisement in The Local Sun, published on the 15 August 2019 and D2 was Editorial paper published in Pen Technology on 1 March 1995. The prior art documents had no drawings.

The claim set which was filed with the description included eight claims, with two independent claims directed at an erasable ink composition and one independent claim directed at a pen. Several other claims were introduced during later statements.

Substantive topics in part 4 included clarity, support by the description, novelty, inventive step, and support for amendments. Of particular interest were:

  • Q16, which contained statements on whether certain features were described as essential in the description. Here, statement 16.3 lacked a reference to a specific claim, by which both T and F could be argued. This statement was neutralized by the Examination Board.

  • Q17, which contained statement 17.4 on the allowability of amendment of a claim in which several ranges are combined. These ranges were individually disclosed in the application as filed but arguments could be made (and were made in candidates in blog comments) that there is no disclosure of this specific combination. This statement was not neutralized by the Examination Board.

  • Q19, which contained statements on whether certain features or effects were ‘mentioned’, ‘disclosed’ or ‘taught’ by prior art documents (and where it appears from the Examiner’s Report that there were no differences intended between the phrasings ‘mentioned’ and ‘disclosed’ and that ‘taught’ also meant ‘disclosed’ but for a technical effect).

  • Q20, which contained statements on whether, in an opposition procedure, certain attacks (novelty, Art. 83 EPC, Art. 123(2) EPC) would be successful (and where it appears from the Examiner’s report that the fact that this took place during an opposition procedure did not appear to be of relevance on the substance of the statements).

  • Statements 18.3 and 20.1, where the answer to each respective statement was dependent on whether one considered D2 to disclose specific ranges of “water” in view of D2’s literal disclosure of ranges of a “water-based solvent system” (and without indication in the paper on how much water was present in this water-based solvent system). Despite this apparent discrepancy between the claim and the disclosure of D2, the statements were not neutralized. See also below for questions asked during the Tutor Meeting on this topic.

Comments and questions at the Tutor Meeting

Rather than showing a presentation with all Pre-Exam questions one-by-one, Stefan Götsch, Joanne Moore and Volker Franz went through the general questions that were submitted in writing prior to the meeting.

In a first set of questions, a tutor asked whether the Examination Board had checked the MC questions of Pre-Exam 2023, whether there had been guineapig testing, whether the Examination Board promotes re-use of earlier Pre-Exam questions and whether the latter are properly checked and updated (while the reference to the former 24 month time limit was kept when re-using the divisional question despite that 24 month clause having been abolished already in 2014). The tutor also asked whether also ion future exams there will be a re-use of earlier questions.

The Committee answered that all questions are prepared by EC-IV (Pre-Exam committee), and all are checked and approved by the Examination Board. Also, there was guineapig testing with other Committee members from other Exam Committees.

Re-used questions are checked in same way as newly generated questions, including testing with guineapigs and review by the Examination Board. There are no instructions that would prevent re-use. EC-IV was aware that the 24 month provision abolished in 2014 but decided to keep the statement without amendment as it tests that candidates are aware that there were no further requirements than pendency. Future exams will again re-use questions, which may also be based on former legal provisions and which will be updated where necessary and appropriate.

The same tutor also noted that the Examiner’s Report was initially published on the Compendium webpage (8 May), then corrected and expanded and a new version was uploaded on the Compendium webpage (10 July) replacing the initial version and without indicating that this was a new/correct version. The tutor asked whether the Examination Board had approved this, and requested that when a new Examiner’s Report is published, as e.g. in 2018 for paper A, an indication “Rev. 1” is given and the original version is maintained.

The Committee indicated that they always include version number.

Joanne Moore commented to general questions submitted by two other tutors:

Q: Several questions were reused from earlier Pre-Exams, with some changes made to have them slightly different, which sometimes introduced some potential unclarities. Some examples are:

Q-a: E.g., Q.1 corresponded to Q.1 of Pre-Exam 2019, with the names of the persons and dates adapted. This resulted in the date indicated as “Today” (6 March 2023) to be different from the date of the exam (17 March 2023) – which was critical for being in time or not.

A-a: Committee wants to remind candidates that they should use the information in the question, and thus use 6/3/23 as today if thew question says so. However, the Committee has decided to avoid confusion with the today-date in the future.

Q-b: Q.3 corresponded to Q.5 of, again, Pre-Exam 2019, with the names, nationalities and residencies of the persons adapted. As a result, one of the applicant now lived in France, introducing a complication due to the national requirement of France to first file with the French office. Q.7 has a similar complication in relation to PCT Rule 19.4(a)(ii), where some candidates considered that the nation al provisions of France could/would prevent the INPI from transmitting the PCT application to the IB. This made some candidates considered that these provisions could/would prevent the valid filing of EP-S with the EPO, or PCT-S with the EPO or the IB. The original question did not have this complication. In this context, it is noted that the 2023 versions of the GL/EPO, GL/PCT-EPO, AG-IP and Euro-PCT Guide are all silent about the effect of national security restrictions in the situations of Q.3 and Q.7.

A-b: The Committee answered that nationalities and national restrictions did not make difference to the answer as EPO would anyhow accord the filing date. Unfortunately, the Committee did not address the issue as to how candidates could be aware of this.

Q-c: Q.6 corresponded to Q.2 of Pre-Exam 2014, with the dates adapted. As for the exam of 2024, the previous version of R.36(1) EPC was in force, the original exam tested that 24m clause. However, the current exam also had a statement reading “An EP divisional application cannot be validly filed after 24 months from the issuance of the first communication under Article 94(3) EPC in respect of EP-P and therefore EP-D was not validly filed as a divisional application.”! It seems not appropriate to confuse candidates with provisions that was only in force until 01.04.2014 (even so long ago that there would not even be any candidates that have the rule still in their reference material): in view of the tutor, the statement could better have been amended to relate to the current law.

A-c: The statement is not incorrect and can be unambiguously answered; it is checking the conditions for filing a divisional.

Q: The Pre-Exam paper became available on the Compendium only in Wiseflow, and not as a pdf-file. It is kindly requested to make each paper (Pre-Exam, D, A, B, C) available as pdf on the EQE Compendium pages 1 hour after the end of each respective exam paper, so that they are available for all candidates as well as tutors.

A: The exam paper is only available in Wiseflow. But a pdf version of paper will be requested.

Q: Already on the day of the exam, candidates ask tutors and on the blogs when the results will be available. It is requested that the Instructions to the candidates for Pre-Exam 2024 already indicate when the exam answers (Examiner’s Report) as well as when the exam results (scores) can be expected. If possible by date, else by week.

A: The finalization and availability of the Examiner’s Report depends on checks and controls take time. It is not possible to get the Examiner’s Report available earlier.

Q: Randomizing the sequence of statements within a single question in the legal part and in the claims part does not provide an equal level playing field (whereas equal treatment is of key importance according to D 11/19), as some candidates got “easier” sequences then others. It is therefore requested to provide the same sequence of questions statements to all candidates, in the legal as well as in the claims parts, in order to provide an equal-levelled playing field.

A: For paper exams where all candidates received the same paper exam with the questions and statements in the same order for all candidates, the order of questions and statements was sometimes deliberate. The online exam is nowadays designed such that sequence does not matter, such that randomization can be done.

Q: As a result of the neutralization in Pre-Exam 2022 of complete part 3 (Q.11-15) as well as Q.20, i.e., for a total of 30 out of 100 marks, the pass rate for that Pre-Exam was as high as 97% (658 of 680 candidates passed; 22 did not pass. How many of these candidates re-sat the Pre-Exam in 2023? How many of these passed in 2023 / how many failed again?

A: The Committee has no information on this.

Stefan Götsch commented to questions on specific statements from these two other tutors:

Q: In Q.5, the time limit for entry into the European phase, after withdrawal of priority claim, had to be calculated: 31/1/22 + 31m [R.159(1) EPC] -> 31/8/2024 (Sat) [R.134(1) EPC] -> 2/9/24. Hence, a calendar of 2024 was needed to come to the correct answer. According to the Examiner’s Report, “This statement has been neutralized, because the calendar for 2024 was not made available to the candidates”. However, no calendars were made available at all, in line with the Notice from the Examination Board of 19 November 2021. It is also noted that the calendar for 2023 was only made available in the OJ 2022, A107, published 30.11.2022, i.e., after the legal cut-off date of 31.10.2022. So, also for any time limits expiring in the year of the exam, no closure dates are covered by the syllabus. This case shows that the non-availability of the required calendars for EPO closure dates, in combination with not being able to consult a calendar for Saturdays and Sundays, leads to unwanted effects. Therefore, it is requested to, as of EQE 2024, again provide calendars showing the EPO closure dates as well as the Saturdays and Sundays with the exam paper, e.g., as external legal resource.

A: The calendar for 2024 was needed to correctly answer Q.5, which candidates did not have available and was neutralized. It is the intention of the Committee to provide calendars to the candidates before the exam; whether this is possible will be checked with the EQE Secretariat.’

Q: The case statement of Q.5 provides: “On 31 January 2022, Susanne […] filed a PCT application PCT-S in German, validly claiming priority from her European patent application EP-S […].” Statement 5.4 provides: “Withdrawing the priority claim of EP-S will affect the effective date of claim 2.” For which the Examiner’s Report provides answer: “FALSE: Claim 2 was not disclosed in EP-S, the priority is not valid for this claim, and the effective date of this claim is the filing date of PCT-S (Article 88(3) EPC).” So, a candidates was expected to determine whether priority was valid or not before the withdrawal (not) and after (also not). However, the term “validly claiming priority” in the case statement may have led candidates to skip the first check, as the case could be understood to already give as a fact that priority was valid, for all claims, so also for claim 2. Presumably, the drafters of the question have meant the term “validly claiming priority” to indicate that the priority declaration / priority claim satisfied (just) the formal requirements for such priority claim (PCT Art. 8, Rule 4.10 referring to Art. 4 Paris Convention; PCT R.26bis.1), while not intending to imply whether priority is actually valid for the claimed subject-matter. It is requested to refrain from using terms that may easily be misunderstood

A: The Committee always tries to use phrasing that cannot be misunderstood. Here, the term is also used in the GL, so clear what is meant.

Q: Statement 14.4 provides (emphasis added): “A valid argument that the cleaning utensil of claim III.1 involves an inventive step is that the claimed ranges of the fibre portions are not obvious from D1 since D1 gives the skilled person no hint at a technical advantage associated with a larger portion of lighter fibres so that the skilled person would not increase the portion of the lighter fibres to more than the maximum 64% disclosed in D1.” For which the Examiner’s Report provides (emphasis added): “TRUE: Indeed, D1 does not disclose any technical effect associated with respect to the amount of the lighter fibres in its cleaning cloth. Thus, in the present case this is one example of a valid argument to support the inventive step of claim III.1.”

The answer clearly indicates that there could be more examples of valid arguments than just this one. Indeed, more arguments may be needed to argue that claim III.1 is indeed inventive over D1 (e.g., in relation to the ends/loops), but in an inventive step argumentation, the argument given in statement 14.4 is just one of a series of arguments that together form a complete reasoning in support of inventive step. 1) Can the Pre-Exam committee indicate what they consider “a valid argument”; 2) Is it any argument that can be part of a complete reasoning? Or only the “key arguments” thereof; 3) Can an argument be “a valid argument” even it is by itself not a sufficient argument to demonstrate inventive step?

A: The term “valid argument” is used more or less each year. A “valid argument” is an argument that is not faulty, that has no flaws. So, is a specific argument that is considered valid. It is not necessarily complete. It is not necessarily successful.

Q: The neutralization in Q.16 is understood and appreciated. Can the Committee indicate how former exams are evaluated, including complaints and appeals, and how that evaluation is used in the design and review of new exams? Is there a plan-to-check-action cycle?

A: The Committee uses evaluation of former exams, complaints and appeals: the Committee evaluates them and uses them in the design of the exam. Direct feedback from candidates during pre-exam is absent, so the Committee uses all other input for drafting of questions, such as these meetings.

Q: W.r.t. statement 17.4 (which was not neutralized), one could argue that while the specific range for the amount of water is explicitly disclosed in [009] of the application, this range is described in relation to “some implementations” and thereby not sufficiently linked to the embodiment represented by claim I.8. Likewise, if the client’s application were to be considered for support for claim I.8bis, one could argue that while each of the ranges claimed in claim I.8bis is disclosed individually in the client’s application, the specific combination is not suggested. There are thus also valid arguments in support of the amendment violating Art. 123(2) EPC and answering ‘False’ to this statement. Can the Committee and the Examination Board indicate why the statement was not neutralized?

A: Statement 17.4 was not neutralized, as the Committee considers the answer sufficiently clear.

Q: In assessing statements 18.3 and 20.1, the Committee appears to consider D2 to disclose specific ranges of water while D2 just discloses ranges of “a water-based solvent system”. However, when considering that a certain percentage of a water-based solvent system does not directly and unambiguously disclose a certain percentage of water (as the water-based solvent system may, or even will, have additional components than just water), the answers to 18.3 and 20.1 flip. See below for details for the two statements, 18.3 and 20.1. Neutralization of these statements was however not done.

Q-a): In assessing statement 18.3, the Committee appears to consider D2 to anticipate the subject matter of claim I.3. However, there are valid arguments to be made that D2 in fact does not anticipate claim I.3. Namely, according to D2, the “solvent system” of D2 is “water-based” and does not directly and unambiguously disclose any amount of water: while a range is given for the solvent system, it is not clear how much of the solvent system is actually water. Without specialist knowledge in this field (and possibly even with such knowledge), it stands to reason that a water-based solvent system comprises additional components besides water. This follows from the plain meaning of the term “water-based” which normally indicates more constituents than just water and, even more, the term “system” which normally indicates a group of components. If this interpretation is followed, there is necessarily less water present in the erasable ink composition of D2 than indicated by the numerical values given for the solvent system itself. Since the amount of water is not otherwise disclosed, one cannot directly and unambiguously conclude whether the disclosure of the water-based solvent system of D2 anticipates the claimed water range in claim I.3.

In this respect, it is noted that the Examiner’s Report refers to the client’s application as providing a definition that “water-based solvent system” means that the solvent system consists of (i.e., is 100%) waterThe Examiner’s Report provides, in the argumentation for 18.3: “The description in [002] of the application mentions that water-based ink are based on using water as a solvent rather than using other solvents. This means that “water-based” corresponds to “water”.”. However, it was argued that this argument is not convincing for the following reasons:

  • [002] of the client’s application simply states that “water-based inks are based on using water as a solvent” which does not go beyond the direct meaning of the term “water-based”, i.e., that the solvent system is based on water. Also, the phrasing “using water … as a solvent” does not indicate that a “solvent system”, which is the actual claim feature in claim I.3, does not comprise other components.
  • Furthermore, the sentence in [002] of the description of the application cannot be used to interpret the prior art differently than what would be the common interpretation: one cannot redefine the prior art in one’s own application and thereby create novelty. Hence, when applying the gold standard to the prior art disclosure, one can argue that there is no unambiguous disclosure of an amount of water (only) which could anticipate the ranges in claim.

However, the statement was not neutralized by the Examination Board, while it was mentioned before that statements would be realized in case of doubt and ambiguities about their interpretation or correct answer.

Can the Committee and the Examination Board indicate why the statement was not neutralized?

Q-b): For Q.20.1, the Examiner’s Report argues “True”. No neutralization was done. However, as for statement 18.3, one may argue also here that one cannot conclude whether the disclosure of the water-based solvent system of D2 anticipates the claimed water range in claim II.1.

Another argument against D2 anticipating the claim is that claim II.1 claims a specific combination of ranges. However, there is no direct and unambiguous disclosure of one embodiment in D2 which anticipates all of these ranges at once. Rather, D2 itself discloses various ranges (which have end points which do fall within the claimed ranges) but these ranges in D2 are not part of a same embodiment. One could thus argue that claim II.1 is novel in view of the “two-list” selection principle of Guidelines (2022) G-VI 8.

Given the above, one may argue that a novelty attack based on D2 will fail. However, the statement was not neutralized by the Examination Board. Can the Committee and the Examination Board indicate why the statement was not neutralized?

A to Q-a) and Q-b): When setting up the Examiner’s Report, the Committee considered the answer sufficiently clear. The Committee does not know of candidates that have been failed because of these two questions (note: some did, as is known from the blogs and from private communications).

Q: As follow-up, a tutor commented that he was also surprised that 18.3 and 20.1 were not neutralized in view of the earlier information from the Committee that “the disadvantage of the present form of the pre-exam is the need to create ‘black-and-white’ situations, and in which it is only possible to neutralize statements, for example if very good arguments exist for another answer; in such cases, the Committee will neutralize the respective statements”“Tutors’ Report on the EQE 2019 Papers and the Meeting between Tutors and EQE Committees”, Nico Cordes (NL), Luis Ferreira (PT), Andrew Hards (DE), Katerina Hartvichova (CZ), Harrie Marsman (NL), Sander van Rijnswou (NL), and Roel van Woudenberg (NL), epi Information 4/2019., as well as that “the Committee and Examination Board consider statistics, blog posts and comments when deciding on a possible neutralization of a statement or question, as these may indicate possible problems and ambiguities”“Tutors’ Report on the EQE 2021 Papers and the Meeting Between Tutors and EQE Committees”, Nico Cordes (NL), Luis Ferreira (PT), Anna Valborg Guðmundsdóttir (IS), Andrew Hards (DE), Jelle Hoekstra (NL), Harrie Marsman (NL), and Roel van Woudenberg (NL), epi Information 4/2021.“Tutors’ Report on the EQE 2022 Papers and the Meeting between Tutors and EQE Committees”, Nico Cordes (NL), Luis Ferreira (PT), Anna Valborg Guðmundsdóttir (IS), Andrew Hards (DE), Harrie Marsman (NL), Zsofia Pintz (HU), Sander van Rijnswou (NL), and Roel van Woudenberg (NL), epi Information 4/2022. - that these statements were widely discussed on the blogs with good and detailed arguments seems a reason to expect neutralization.

A: The Committee sees this as a clear definition of the term “water-based”, and not just a term in the application. Volker suggested to agree to disagree, and the discussion was closed.

Volker expressed his thanks for questions asked by tutors, as they are useful for thinking questions for the future.

9) Concluding remarks

The annual meeting of EQE tutors and members of the EQE Committees and of the Examination Board took place on 18 and 19 October 2023. The meeting took place by videoconference.

With this report, tutors summarize the papers and provide information of the points discussed at the meeting so that candidates and other tutors can also find this information. In addition, we hope that our summaries and comments can assist when reading and interpreting the official Examiners’ ReportsCandidates are reminded that “The purpose of the examiner’s report is to enable candidates to prepare for future examinations (cf. Art 6(6) of the Regulations on the European qualifying examination for professional representatives).” The Examiners’ Reports are available in the Compendium on the EQE website (https://www.epo.org/en/learning/professional-hub/european-qualifying-examination-eqe/compendium). of the EQE 2023 papers.



    Comments